<< Chapter < Page Chapter >> Page >
net τ = 0 size 12{"net "τ=0} {}

where net means total. Torques, which are in opposite directions are assigned opposite signs. A common convention is to call counterclockwise (ccw) torques positive and clockwise (cw) torques negative.

When two children balance a seesaw as shown in [link] , they satisfy the two conditions for equilibrium. Most people have perfect intuition about seesaws, knowing that the lighter child must sit farther from the pivot and that a heavier child can keep a lighter one off the ground indefinitely.

Two children are sitting on a seesaw. On the left side, a lighter child is sitting and on the right, a heavier one. The distance of the lighter child from the fulcrum is more than that of heavier child. At the fulcrum, an upward force vector labeled as F-p is shown. The weights of the two children, w-one and w-two, are shown as vectors in the downward direction, and the force at the fulcrum, F-p, is shown as a vector in the upward direction.
Two children balancing a seesaw satisfy both conditions for equilibrium. The lighter child sits farther from the pivot to create a torque equal in magnitude to that of the heavier child.

She saw torques on a seesaw

The two children shown in [link] are balanced on a seesaw of negligible mass. (This assumption is made to keep the example simple—more involved examples will follow.) The first child has a mass of 26.0 kg and sits 1.60 m from the pivot.(a) If the second child has a mass of 32.0 kg, how far is she from the pivot? (b) What is F p , the supporting force exerted by the pivot?

Strategy

Both conditions for equilibrium must be satisfied. In part (a), we are asked for a distance; thus, the second condition (regarding torques) must be used, since the first (regarding only forces) has no distances in it. To apply the second condition for equilibrium, we first identify the system of interest to be the seesaw plus the two children. We take the supporting pivot to be the point about which the torques are calculated. We then identify all external forces acting on the system.

Solution (a)

The three external forces acting on the system are the weights of the two children and the supporting force of the pivot. Let us examine the torque produced by each. Torque is defined to be

τ = rF sin θ . size 12{τ= ital "rF""sin"θ} {}

Here θ = 90º , so that sin θ = 1 for all three forces. That means r = r size 12{r rSub { size 8{ ortho } } =r} {} for all three. The torques exerted by the three forces are first,

τ 1 = r 1 w 1

second,

τ 2 = r 2 w 2

and third,

τ p = r p F p = 0 F p = 0.

Note that a minus sign has been inserted into the second equation because this torque is clockwise and is therefore negative by convention. Since F p acts directly on the pivot point, the distance r p is zero. A force acting on the pivot cannot cause a rotation, just as pushing directly on the hinges of a door will not cause it to rotate. Now, the second condition for equilibrium is that the sum of the torques on both children is zero. Therefore

τ 2 = τ 1 , size 12{τ rSub { size 8{2} } =τ rSub { size 8{1} } } {}

or

r 2 w 2 = r 1 w 1 . size 12{r rSub { size 8{2} } w rSub { size 8{2} } =r rSub { size 8{1} } w rSub { size 8{1} } } {}

Weight is mass times the acceleration due to gravity. Entering mg for w , we get

r 2 m 2 g = r 1 m 1 g .

Solve this for the unknown r 2 size 12{r rSub { size 8{2} } } {} :

r 2 = r 1 m 1 m 2 . size 12{r rSub { size 8{2} } =r rSub { size 8{1} } { {m rSub { size 8{1} } } over {m rSub { size 8{2} } } } } {}

The quantities on the right side of the equation are known; thus, r 2 size 12{r rSub { size 8{2} } } {} is

r 2 = 1.60 m 26.0 kg 32.0 kg = 1.30 m . size 12{r rSub { size 8{2} } = left (1 "." "60"" m" right ) { {"26" "." 0" kg"} over {"32" "." " 0 kg"} } =1 "." "30"" m"} {}

As expected, the heavier child must sit closer to the pivot (1.30 m versus 1.60 m) to balance the seesaw.

Solution (b)

This part asks for a force F p . The easiest way to find it is to use the first condition for equilibrium, which is

net F = 0 .

The forces are all vertical, so that we are dealing with a one-dimensional problem along the vertical axis; hence, the condition can be written as

Practice Key Terms 4

Get Jobilize Job Search Mobile App in your pocket Now!

Get it on Google Play Download on the App Store Now




Source:  OpenStax, Unit 7 - statics and torque. OpenStax CNX. Feb 03, 2016 Download for free at https://legacy.cnx.org/content/col11967/1.1
Google Play and the Google Play logo are trademarks of Google Inc.

Notification Switch

Would you like to follow the 'Unit 7 - statics and torque' conversation and receive update notifications?

Ask